Finding relationship between solutions of two different first order linear PDEs (using Feynman-Kac)Existence of first order PDEs?Relationship of SDE and Feynman-Kac PDECharacteristic Function of Geometric Brownian motion - PDE with terminal condition approachDifference Between Feynman-Kac PDE and Kolmogorov Backward EquationUse a stochastic representation result (feynman kac theorem) to solve boundary value problemStochastic Representation of Boundary Value ProblemEntire solutions of first order linear homogeneous evolution PDEsCritical points of solutions to linear PDEsStochastic differential equation with quadratic drift and volatilityCoupled first order PDEs using characteristics

Landlord wants to switch my lease to a "Land contract" to "get back at the city"

Some basic questions on halt and move in Turing machines

Can I legally use front facing blue light in the UK?

Piano - What is the notation for a double stop where both notes in the double stop are different lengths?

Patience, young "Padovan"

I’m planning on buying a laser printer but concerned about the life cycle of toner in the machine

Doomsday-clock for my fantasy planet

Copycat chess is back

Latin words with no plurals in English

Mapping arrows in commutative diagrams

Lied on resume at previous job

Need help identifying/translating a plaque in Tangier, Morocco

Pristine Bit Checking

Ideas for colorfully and clearly highlighting graph edges according to weights

Is Social Media Science Fiction?

How to manage monthly salary

How did the USSR manage to innovate in an environment characterized by government censorship and high bureaucracy?

Shall I use personal or official e-mail account when registering to external websites for work purpose?

Is there a way to make member function NOT callable from constructor?

How is it possible for user's password to be changed after storage was encrypted? (on OS X, Android)

Is Fable (1996) connected in any way to the Fable franchise from Lionhead Studios?

Domain expired, GoDaddy holds it and is asking more money

Why do we use polarized capacitors?

Can produce flame be used to grapple, or as an unarmed strike, in the right circumstances?



Finding relationship between solutions of two different first order linear PDEs (using Feynman-Kac)


Existence of first order PDEs?Relationship of SDE and Feynman-Kac PDECharacteristic Function of Geometric Brownian motion - PDE with terminal condition approachDifference Between Feynman-Kac PDE and Kolmogorov Backward EquationUse a stochastic representation result (feynman kac theorem) to solve boundary value problemStochastic Representation of Boundary Value ProblemEntire solutions of first order linear homogeneous evolution PDEsCritical points of solutions to linear PDEsStochastic differential equation with quadratic drift and volatilityCoupled first order PDEs using characteristics













0












$begingroup$


The first partial differential equation, using the shorthand $mathcalLu(t,x) = mu(x,t) fracpartial upartial x(x,t) + tfrac12 sigma^2(x,t) fracpartial^2 upartial x^2(x,t)$, is:
beginequation
fracpartial upartial t(x,t) + mathcalLu(x,t) -Vu(x,t) + f(x,t) = 0
tag1endequation



and the other is
beginequation
fracpartial vpartial t(x,t) + mathcalLv(x,t) -(V + W)* v(x,t) + f(x,t) = 0
tag2
endequation



both with terminal condition: $v(T,x) = u(T,x) = g(x)$ and with constants $V$ and $W$.



So the only difference is that the second equation has the term $-Wv(x,t)$ included. I want to find out how I can relate the two solutions, so that if I have a solution of $(1)$ I can multiply it by something and get a solution of $(2)$.



I am wondering if this is possible. So far, I have written, using Feynman-Kac, solutions to both equations:



beginequation
u(x,t) = E^Qleft[ int_t^T e^-V (r-t)f(X_r,r)dr + e^-V(T-t)g(X_T) Bigg| X_t=x right]
endequation



beginequation
v(x,t) = E^Qleft[ int_t^T e^-(V +G) (r-t)f(X_r,r)dr + e^-(V+G)(T-t)g(X_T) Bigg| X_t=x right]
endequation



I can't seem to think of a way to connect the two together, but according to a book I'm reading there should be a way to do this (but there's no other details). Any help would be greatly appreciated. Thanks!










share|cite|improve this question









$endgroup$
















    0












    $begingroup$


    The first partial differential equation, using the shorthand $mathcalLu(t,x) = mu(x,t) fracpartial upartial x(x,t) + tfrac12 sigma^2(x,t) fracpartial^2 upartial x^2(x,t)$, is:
    beginequation
    fracpartial upartial t(x,t) + mathcalLu(x,t) -Vu(x,t) + f(x,t) = 0
    tag1endequation



    and the other is
    beginequation
    fracpartial vpartial t(x,t) + mathcalLv(x,t) -(V + W)* v(x,t) + f(x,t) = 0
    tag2
    endequation



    both with terminal condition: $v(T,x) = u(T,x) = g(x)$ and with constants $V$ and $W$.



    So the only difference is that the second equation has the term $-Wv(x,t)$ included. I want to find out how I can relate the two solutions, so that if I have a solution of $(1)$ I can multiply it by something and get a solution of $(2)$.



    I am wondering if this is possible. So far, I have written, using Feynman-Kac, solutions to both equations:



    beginequation
    u(x,t) = E^Qleft[ int_t^T e^-V (r-t)f(X_r,r)dr + e^-V(T-t)g(X_T) Bigg| X_t=x right]
    endequation



    beginequation
    v(x,t) = E^Qleft[ int_t^T e^-(V +G) (r-t)f(X_r,r)dr + e^-(V+G)(T-t)g(X_T) Bigg| X_t=x right]
    endequation



    I can't seem to think of a way to connect the two together, but according to a book I'm reading there should be a way to do this (but there's no other details). Any help would be greatly appreciated. Thanks!










    share|cite|improve this question









    $endgroup$














      0












      0








      0


      1



      $begingroup$


      The first partial differential equation, using the shorthand $mathcalLu(t,x) = mu(x,t) fracpartial upartial x(x,t) + tfrac12 sigma^2(x,t) fracpartial^2 upartial x^2(x,t)$, is:
      beginequation
      fracpartial upartial t(x,t) + mathcalLu(x,t) -Vu(x,t) + f(x,t) = 0
      tag1endequation



      and the other is
      beginequation
      fracpartial vpartial t(x,t) + mathcalLv(x,t) -(V + W)* v(x,t) + f(x,t) = 0
      tag2
      endequation



      both with terminal condition: $v(T,x) = u(T,x) = g(x)$ and with constants $V$ and $W$.



      So the only difference is that the second equation has the term $-Wv(x,t)$ included. I want to find out how I can relate the two solutions, so that if I have a solution of $(1)$ I can multiply it by something and get a solution of $(2)$.



      I am wondering if this is possible. So far, I have written, using Feynman-Kac, solutions to both equations:



      beginequation
      u(x,t) = E^Qleft[ int_t^T e^-V (r-t)f(X_r,r)dr + e^-V(T-t)g(X_T) Bigg| X_t=x right]
      endequation



      beginequation
      v(x,t) = E^Qleft[ int_t^T e^-(V +G) (r-t)f(X_r,r)dr + e^-(V+G)(T-t)g(X_T) Bigg| X_t=x right]
      endequation



      I can't seem to think of a way to connect the two together, but according to a book I'm reading there should be a way to do this (but there's no other details). Any help would be greatly appreciated. Thanks!










      share|cite|improve this question









      $endgroup$




      The first partial differential equation, using the shorthand $mathcalLu(t,x) = mu(x,t) fracpartial upartial x(x,t) + tfrac12 sigma^2(x,t) fracpartial^2 upartial x^2(x,t)$, is:
      beginequation
      fracpartial upartial t(x,t) + mathcalLu(x,t) -Vu(x,t) + f(x,t) = 0
      tag1endequation



      and the other is
      beginequation
      fracpartial vpartial t(x,t) + mathcalLv(x,t) -(V + W)* v(x,t) + f(x,t) = 0
      tag2
      endequation



      both with terminal condition: $v(T,x) = u(T,x) = g(x)$ and with constants $V$ and $W$.



      So the only difference is that the second equation has the term $-Wv(x,t)$ included. I want to find out how I can relate the two solutions, so that if I have a solution of $(1)$ I can multiply it by something and get a solution of $(2)$.



      I am wondering if this is possible. So far, I have written, using Feynman-Kac, solutions to both equations:



      beginequation
      u(x,t) = E^Qleft[ int_t^T e^-V (r-t)f(X_r,r)dr + e^-V(T-t)g(X_T) Bigg| X_t=x right]
      endequation



      beginequation
      v(x,t) = E^Qleft[ int_t^T e^-(V +G) (r-t)f(X_r,r)dr + e^-(V+G)(T-t)g(X_T) Bigg| X_t=x right]
      endequation



      I can't seem to think of a way to connect the two together, but according to a book I'm reading there should be a way to do this (but there's no other details). Any help would be greatly appreciated. Thanks!







      pde stochastic-calculus stochastic-pde






      share|cite|improve this question













      share|cite|improve this question











      share|cite|improve this question




      share|cite|improve this question










      asked Mar 30 at 2:52









      SladeSlade

      66111




      66111




















          1 Answer
          1






          active

          oldest

          votes


















          1












          $begingroup$

          Suppose $u$ and $v$ solve
          $$
          left[partial_t+mathcalL-cright]u+f=0
          qquad textand qquad
          left[partial_t+mathcalL-left(c+dright)right]v+e^dtf=0.
          $$

          Moreover, suppose $u(T,cdot)=g$ and $v(T,cdot)=e^dTg$. If we
          assume enough regularity on $u$ and $v$ along with $mu$, $sigma$,
          and $g$, the Feynman-Kac formula tells us that
          $$
          u(t,x)=mathbbEleft[int_t^Te^-c(r-t)f(r,X_r)dr+e^-cleft(T-tright)g(X_T)mid X_t=xright]
          $$

          and
          beginalign*
          v(t,x) & =mathbbEleft[int_t^Te^-(c+d)(r-t)e^drf(r,X_r)dr+e^-(c+d)(T-t)e^dTg(X_T)right]\
          & =mathbbEleft[int_t^Te^-c(r-t)f(r,X_r)dr+e^-c(T-t)g(X_T)right]e^dt\
          & =e^dtu(t,x)
          endalign*

          where $X$ satisfies an Ito SDE with infinitesimal generator $mathcalL$



          Are you sure this is not what you/the text meant?



          Note: a simpler argument to go from one PDE to the other is by a change of variables.






          share|cite|improve this answer









          $endgroup$








          • 1




            $begingroup$
            This was it! Thanks a lot! The text was unclear so I didn't realize the $f$ term could be adjusted as well. Can you elaborate on the change of variables method? Is it just using $left[partial_t+mathcalL-left(c+dright)right]v+e^dtf=0$ with $v = e^dtu(t,x)$ and then expanding out $partial_tv = partial_te^dtu = partial_tu * e^dt + d*e^dtu$, and so this results in $left[partial_t+mathcalL-cright]u+f=0$ when dividing out $e^dt$. (Or doing the reverse by multiplying the $u$ equation by $e^dt$.)
            $endgroup$
            – Slade
            Mar 30 at 13:01










          • $begingroup$
            That's right (+1). If $u$ is a $C^1,2$ solution of the first PDE, then you can define $v equiv e^dt u$ to be a solution of the second. Vice versa, if $v$ is a $C^1,2$ solution of the second PDE, you can define $u equiv e^-dt v$ as a solution of the first.
            $endgroup$
            – parsiad
            Mar 30 at 19:33











          • $begingroup$
            Nit: Try not to use $*$ to mean multiplication. This operator is commonly used for convolutions.
            $endgroup$
            – parsiad
            Mar 30 at 19:34











          Your Answer





          StackExchange.ifUsing("editor", function ()
          return StackExchange.using("mathjaxEditing", function ()
          StackExchange.MarkdownEditor.creationCallbacks.add(function (editor, postfix)
          StackExchange.mathjaxEditing.prepareWmdForMathJax(editor, postfix, [["$", "$"], ["\\(","\\)"]]);
          );
          );
          , "mathjax-editing");

          StackExchange.ready(function()
          var channelOptions =
          tags: "".split(" "),
          id: "69"
          ;
          initTagRenderer("".split(" "), "".split(" "), channelOptions);

          StackExchange.using("externalEditor", function()
          // Have to fire editor after snippets, if snippets enabled
          if (StackExchange.settings.snippets.snippetsEnabled)
          StackExchange.using("snippets", function()
          createEditor();
          );

          else
          createEditor();

          );

          function createEditor()
          StackExchange.prepareEditor(
          heartbeatType: 'answer',
          autoActivateHeartbeat: false,
          convertImagesToLinks: true,
          noModals: true,
          showLowRepImageUploadWarning: true,
          reputationToPostImages: 10,
          bindNavPrevention: true,
          postfix: "",
          imageUploader:
          brandingHtml: "Powered by u003ca class="icon-imgur-white" href="https://imgur.com/"u003eu003c/au003e",
          contentPolicyHtml: "User contributions licensed under u003ca href="https://creativecommons.org/licenses/by-sa/3.0/"u003ecc by-sa 3.0 with attribution requiredu003c/au003e u003ca href="https://stackoverflow.com/legal/content-policy"u003e(content policy)u003c/au003e",
          allowUrls: true
          ,
          noCode: true, onDemand: true,
          discardSelector: ".discard-answer"
          ,immediatelyShowMarkdownHelp:true
          );



          );













          draft saved

          draft discarded


















          StackExchange.ready(
          function ()
          StackExchange.openid.initPostLogin('.new-post-login', 'https%3a%2f%2fmath.stackexchange.com%2fquestions%2f3167870%2ffinding-relationship-between-solutions-of-two-different-first-order-linear-pdes%23new-answer', 'question_page');

          );

          Post as a guest















          Required, but never shown

























          1 Answer
          1






          active

          oldest

          votes








          1 Answer
          1






          active

          oldest

          votes









          active

          oldest

          votes






          active

          oldest

          votes









          1












          $begingroup$

          Suppose $u$ and $v$ solve
          $$
          left[partial_t+mathcalL-cright]u+f=0
          qquad textand qquad
          left[partial_t+mathcalL-left(c+dright)right]v+e^dtf=0.
          $$

          Moreover, suppose $u(T,cdot)=g$ and $v(T,cdot)=e^dTg$. If we
          assume enough regularity on $u$ and $v$ along with $mu$, $sigma$,
          and $g$, the Feynman-Kac formula tells us that
          $$
          u(t,x)=mathbbEleft[int_t^Te^-c(r-t)f(r,X_r)dr+e^-cleft(T-tright)g(X_T)mid X_t=xright]
          $$

          and
          beginalign*
          v(t,x) & =mathbbEleft[int_t^Te^-(c+d)(r-t)e^drf(r,X_r)dr+e^-(c+d)(T-t)e^dTg(X_T)right]\
          & =mathbbEleft[int_t^Te^-c(r-t)f(r,X_r)dr+e^-c(T-t)g(X_T)right]e^dt\
          & =e^dtu(t,x)
          endalign*

          where $X$ satisfies an Ito SDE with infinitesimal generator $mathcalL$



          Are you sure this is not what you/the text meant?



          Note: a simpler argument to go from one PDE to the other is by a change of variables.






          share|cite|improve this answer









          $endgroup$








          • 1




            $begingroup$
            This was it! Thanks a lot! The text was unclear so I didn't realize the $f$ term could be adjusted as well. Can you elaborate on the change of variables method? Is it just using $left[partial_t+mathcalL-left(c+dright)right]v+e^dtf=0$ with $v = e^dtu(t,x)$ and then expanding out $partial_tv = partial_te^dtu = partial_tu * e^dt + d*e^dtu$, and so this results in $left[partial_t+mathcalL-cright]u+f=0$ when dividing out $e^dt$. (Or doing the reverse by multiplying the $u$ equation by $e^dt$.)
            $endgroup$
            – Slade
            Mar 30 at 13:01










          • $begingroup$
            That's right (+1). If $u$ is a $C^1,2$ solution of the first PDE, then you can define $v equiv e^dt u$ to be a solution of the second. Vice versa, if $v$ is a $C^1,2$ solution of the second PDE, you can define $u equiv e^-dt v$ as a solution of the first.
            $endgroup$
            – parsiad
            Mar 30 at 19:33











          • $begingroup$
            Nit: Try not to use $*$ to mean multiplication. This operator is commonly used for convolutions.
            $endgroup$
            – parsiad
            Mar 30 at 19:34















          1












          $begingroup$

          Suppose $u$ and $v$ solve
          $$
          left[partial_t+mathcalL-cright]u+f=0
          qquad textand qquad
          left[partial_t+mathcalL-left(c+dright)right]v+e^dtf=0.
          $$

          Moreover, suppose $u(T,cdot)=g$ and $v(T,cdot)=e^dTg$. If we
          assume enough regularity on $u$ and $v$ along with $mu$, $sigma$,
          and $g$, the Feynman-Kac formula tells us that
          $$
          u(t,x)=mathbbEleft[int_t^Te^-c(r-t)f(r,X_r)dr+e^-cleft(T-tright)g(X_T)mid X_t=xright]
          $$

          and
          beginalign*
          v(t,x) & =mathbbEleft[int_t^Te^-(c+d)(r-t)e^drf(r,X_r)dr+e^-(c+d)(T-t)e^dTg(X_T)right]\
          & =mathbbEleft[int_t^Te^-c(r-t)f(r,X_r)dr+e^-c(T-t)g(X_T)right]e^dt\
          & =e^dtu(t,x)
          endalign*

          where $X$ satisfies an Ito SDE with infinitesimal generator $mathcalL$



          Are you sure this is not what you/the text meant?



          Note: a simpler argument to go from one PDE to the other is by a change of variables.






          share|cite|improve this answer









          $endgroup$








          • 1




            $begingroup$
            This was it! Thanks a lot! The text was unclear so I didn't realize the $f$ term could be adjusted as well. Can you elaborate on the change of variables method? Is it just using $left[partial_t+mathcalL-left(c+dright)right]v+e^dtf=0$ with $v = e^dtu(t,x)$ and then expanding out $partial_tv = partial_te^dtu = partial_tu * e^dt + d*e^dtu$, and so this results in $left[partial_t+mathcalL-cright]u+f=0$ when dividing out $e^dt$. (Or doing the reverse by multiplying the $u$ equation by $e^dt$.)
            $endgroup$
            – Slade
            Mar 30 at 13:01










          • $begingroup$
            That's right (+1). If $u$ is a $C^1,2$ solution of the first PDE, then you can define $v equiv e^dt u$ to be a solution of the second. Vice versa, if $v$ is a $C^1,2$ solution of the second PDE, you can define $u equiv e^-dt v$ as a solution of the first.
            $endgroup$
            – parsiad
            Mar 30 at 19:33











          • $begingroup$
            Nit: Try not to use $*$ to mean multiplication. This operator is commonly used for convolutions.
            $endgroup$
            – parsiad
            Mar 30 at 19:34













          1












          1








          1





          $begingroup$

          Suppose $u$ and $v$ solve
          $$
          left[partial_t+mathcalL-cright]u+f=0
          qquad textand qquad
          left[partial_t+mathcalL-left(c+dright)right]v+e^dtf=0.
          $$

          Moreover, suppose $u(T,cdot)=g$ and $v(T,cdot)=e^dTg$. If we
          assume enough regularity on $u$ and $v$ along with $mu$, $sigma$,
          and $g$, the Feynman-Kac formula tells us that
          $$
          u(t,x)=mathbbEleft[int_t^Te^-c(r-t)f(r,X_r)dr+e^-cleft(T-tright)g(X_T)mid X_t=xright]
          $$

          and
          beginalign*
          v(t,x) & =mathbbEleft[int_t^Te^-(c+d)(r-t)e^drf(r,X_r)dr+e^-(c+d)(T-t)e^dTg(X_T)right]\
          & =mathbbEleft[int_t^Te^-c(r-t)f(r,X_r)dr+e^-c(T-t)g(X_T)right]e^dt\
          & =e^dtu(t,x)
          endalign*

          where $X$ satisfies an Ito SDE with infinitesimal generator $mathcalL$



          Are you sure this is not what you/the text meant?



          Note: a simpler argument to go from one PDE to the other is by a change of variables.






          share|cite|improve this answer









          $endgroup$



          Suppose $u$ and $v$ solve
          $$
          left[partial_t+mathcalL-cright]u+f=0
          qquad textand qquad
          left[partial_t+mathcalL-left(c+dright)right]v+e^dtf=0.
          $$

          Moreover, suppose $u(T,cdot)=g$ and $v(T,cdot)=e^dTg$. If we
          assume enough regularity on $u$ and $v$ along with $mu$, $sigma$,
          and $g$, the Feynman-Kac formula tells us that
          $$
          u(t,x)=mathbbEleft[int_t^Te^-c(r-t)f(r,X_r)dr+e^-cleft(T-tright)g(X_T)mid X_t=xright]
          $$

          and
          beginalign*
          v(t,x) & =mathbbEleft[int_t^Te^-(c+d)(r-t)e^drf(r,X_r)dr+e^-(c+d)(T-t)e^dTg(X_T)right]\
          & =mathbbEleft[int_t^Te^-c(r-t)f(r,X_r)dr+e^-c(T-t)g(X_T)right]e^dt\
          & =e^dtu(t,x)
          endalign*

          where $X$ satisfies an Ito SDE with infinitesimal generator $mathcalL$



          Are you sure this is not what you/the text meant?



          Note: a simpler argument to go from one PDE to the other is by a change of variables.







          share|cite|improve this answer












          share|cite|improve this answer



          share|cite|improve this answer










          answered Mar 30 at 3:50









          parsiadparsiad

          18.7k32453




          18.7k32453







          • 1




            $begingroup$
            This was it! Thanks a lot! The text was unclear so I didn't realize the $f$ term could be adjusted as well. Can you elaborate on the change of variables method? Is it just using $left[partial_t+mathcalL-left(c+dright)right]v+e^dtf=0$ with $v = e^dtu(t,x)$ and then expanding out $partial_tv = partial_te^dtu = partial_tu * e^dt + d*e^dtu$, and so this results in $left[partial_t+mathcalL-cright]u+f=0$ when dividing out $e^dt$. (Or doing the reverse by multiplying the $u$ equation by $e^dt$.)
            $endgroup$
            – Slade
            Mar 30 at 13:01










          • $begingroup$
            That's right (+1). If $u$ is a $C^1,2$ solution of the first PDE, then you can define $v equiv e^dt u$ to be a solution of the second. Vice versa, if $v$ is a $C^1,2$ solution of the second PDE, you can define $u equiv e^-dt v$ as a solution of the first.
            $endgroup$
            – parsiad
            Mar 30 at 19:33











          • $begingroup$
            Nit: Try not to use $*$ to mean multiplication. This operator is commonly used for convolutions.
            $endgroup$
            – parsiad
            Mar 30 at 19:34












          • 1




            $begingroup$
            This was it! Thanks a lot! The text was unclear so I didn't realize the $f$ term could be adjusted as well. Can you elaborate on the change of variables method? Is it just using $left[partial_t+mathcalL-left(c+dright)right]v+e^dtf=0$ with $v = e^dtu(t,x)$ and then expanding out $partial_tv = partial_te^dtu = partial_tu * e^dt + d*e^dtu$, and so this results in $left[partial_t+mathcalL-cright]u+f=0$ when dividing out $e^dt$. (Or doing the reverse by multiplying the $u$ equation by $e^dt$.)
            $endgroup$
            – Slade
            Mar 30 at 13:01










          • $begingroup$
            That's right (+1). If $u$ is a $C^1,2$ solution of the first PDE, then you can define $v equiv e^dt u$ to be a solution of the second. Vice versa, if $v$ is a $C^1,2$ solution of the second PDE, you can define $u equiv e^-dt v$ as a solution of the first.
            $endgroup$
            – parsiad
            Mar 30 at 19:33











          • $begingroup$
            Nit: Try not to use $*$ to mean multiplication. This operator is commonly used for convolutions.
            $endgroup$
            – parsiad
            Mar 30 at 19:34







          1




          1




          $begingroup$
          This was it! Thanks a lot! The text was unclear so I didn't realize the $f$ term could be adjusted as well. Can you elaborate on the change of variables method? Is it just using $left[partial_t+mathcalL-left(c+dright)right]v+e^dtf=0$ with $v = e^dtu(t,x)$ and then expanding out $partial_tv = partial_te^dtu = partial_tu * e^dt + d*e^dtu$, and so this results in $left[partial_t+mathcalL-cright]u+f=0$ when dividing out $e^dt$. (Or doing the reverse by multiplying the $u$ equation by $e^dt$.)
          $endgroup$
          – Slade
          Mar 30 at 13:01




          $begingroup$
          This was it! Thanks a lot! The text was unclear so I didn't realize the $f$ term could be adjusted as well. Can you elaborate on the change of variables method? Is it just using $left[partial_t+mathcalL-left(c+dright)right]v+e^dtf=0$ with $v = e^dtu(t,x)$ and then expanding out $partial_tv = partial_te^dtu = partial_tu * e^dt + d*e^dtu$, and so this results in $left[partial_t+mathcalL-cright]u+f=0$ when dividing out $e^dt$. (Or doing the reverse by multiplying the $u$ equation by $e^dt$.)
          $endgroup$
          – Slade
          Mar 30 at 13:01












          $begingroup$
          That's right (+1). If $u$ is a $C^1,2$ solution of the first PDE, then you can define $v equiv e^dt u$ to be a solution of the second. Vice versa, if $v$ is a $C^1,2$ solution of the second PDE, you can define $u equiv e^-dt v$ as a solution of the first.
          $endgroup$
          – parsiad
          Mar 30 at 19:33





          $begingroup$
          That's right (+1). If $u$ is a $C^1,2$ solution of the first PDE, then you can define $v equiv e^dt u$ to be a solution of the second. Vice versa, if $v$ is a $C^1,2$ solution of the second PDE, you can define $u equiv e^-dt v$ as a solution of the first.
          $endgroup$
          – parsiad
          Mar 30 at 19:33













          $begingroup$
          Nit: Try not to use $*$ to mean multiplication. This operator is commonly used for convolutions.
          $endgroup$
          – parsiad
          Mar 30 at 19:34




          $begingroup$
          Nit: Try not to use $*$ to mean multiplication. This operator is commonly used for convolutions.
          $endgroup$
          – parsiad
          Mar 30 at 19:34

















          draft saved

          draft discarded
















































          Thanks for contributing an answer to Mathematics Stack Exchange!


          • Please be sure to answer the question. Provide details and share your research!

          But avoid


          • Asking for help, clarification, or responding to other answers.

          • Making statements based on opinion; back them up with references or personal experience.

          Use MathJax to format equations. MathJax reference.


          To learn more, see our tips on writing great answers.




          draft saved


          draft discarded














          StackExchange.ready(
          function ()
          StackExchange.openid.initPostLogin('.new-post-login', 'https%3a%2f%2fmath.stackexchange.com%2fquestions%2f3167870%2ffinding-relationship-between-solutions-of-two-different-first-order-linear-pdes%23new-answer', 'question_page');

          );

          Post as a guest















          Required, but never shown





















































          Required, but never shown














          Required, but never shown












          Required, but never shown







          Required, but never shown

































          Required, but never shown














          Required, but never shown












          Required, but never shown







          Required, but never shown







          Popular posts from this blog

          Triangular numbers and gcdProving sum of a set is $0 pmod n$ if $n$ is odd, or $fracn2 pmod n$ if $n$ is even?Is greatest common divisor of two numbers really their smallest linear combination?GCD, LCM RelationshipProve a set of nonnegative integers with greatest common divisor 1 and closed under addition has all but finite many nonnegative integers.all pairs of a and b in an equation containing gcdTriangular Numbers Modulo $k$ - Hit All Values?Understanding the Existence and Uniqueness of the GCDGCD and LCM with logical symbolsThe greatest common divisor of two positive integers less than 100 is equal to 3. Their least common multiple is twelve times one of the integers.Suppose that for all integers $x$, $x|a$ and $x|b$ if and only if $x|c$. Then $c = gcd(a,b)$Which is the gcd of 2 numbers which are multiplied and the result is 600000?

          Ingelân Ynhâld Etymology | Geografy | Skiednis | Polityk en bestjoer | Ekonomy | Demografy | Kultuer | Klimaat | Sjoch ek | Keppelings om utens | Boarnen, noaten en referinsjes Navigaasjemenuwww.gov.ukOffisjele webside fan it regear fan it Feriene KeninkrykOffisjele webside fan it Britske FerkearsburoNederlânsktalige ynformaasje fan it Britske FerkearsburoOffisjele webside fan English Heritage, de organisaasje dy't him ynset foar it behâld fan it Ingelske kultuergoedYnwennertallen fan alle Britske stêden út 'e folkstelling fan 2011Notes en References, op dizze sideEngland

          Հադիս Բովանդակություն Անվանում և նշանակություն | Դասակարգում | Աղբյուրներ | Նավարկման ցանկ